0 of 15 Questions completed
Questions:
You have already completed the quiz before. Hence you can not start it again.
Quiz is loading…
You must sign in or sign up to start the quiz.
You must first complete the following:
0 of 15 Questions answered correctly
Your time:
Time has elapsed
You have reached 0 of 0 point(s), (0)
Earned Point(s): 0 of 0, (0)
0 Essay(s) Pending (Possible Point(s): 0)
Average score |
|
Your score |
|
Which of the following is the mechanism of action of pimavanserin?
A 58-year-old male presents with a few months of progressive symptoms. He reports limb rigidity, left greater than right, as well as neck stiffness. On exam, you note bradykinesia and cervical dystonia. He does not have a tremor. Speech is hesitant, and there is evidence of speech apraxia. You also note that the patient has trouble controlling the location of his left hand in space, and he grabs for your stethoscope without realizing it.
What is the most likely diagnosis?
A 78-year-old male with no past medical history presents to the ED with left-sided hemiparesis and slurred speech at 9 AM. She is unable to provide a clear clinical history. She was noted to be found at 7:30 AM in bed with the above-stated symptoms. Her husband states that the patient went to bed at 10:00 PM the previous night without any symptoms. CT head shows no blood or hypodensities, and CTA head/neck reveals a right MCA occlusion. Her NIHSS is 17. Which of the following treatment options is the most appropriate next step based on the available data?
Which of the following is not part of Gerstmann’s syndrome?
A 21-year-old man is brought into the clinic by his roommate due to bizarre behavior. For the last 10 months, the patient has been intermittently skipping classes and trying to soundproof his dorm room because he believes the government is watching him. His roommate says that on two separate occasions he caught the patient talking to himself under the bed covers. The neurological exam is normal but the patient refuses to make eye contact and his speech is pressured and tangential. Which of the following is the most likely diagnosis?
A neurology consult was placed for a 35-year-old male who was admitted overnight after a motor vehicle accident that occurred earlier in the day. He experienced multiple bone fractures, including the right humerus at the spiral groove. The primary team is concerned about a radial nerve injury. Which of the following findings are most likely to be found on EMG/NCS?
Which of the following medications can potentially reduce the platelet inhibition of clopidogrel?
A 42-year-old woman with major depressive disorder and anxiety who has recently completed treatment for gambling addiction presents with a unilateral upper extremity resting tremor and the feeling that she is “stiff all of the time”. Her husband remarks that she has become much more soft-spoken lately and that five years ago she began acting out her dreams. Which of the following medications should be avoided in this patient?
A 3-year-old female with a past medical history of recurrent sinus infections is brought to the clinic by her father because of unsteadiness on her feet for the past 4 months. Her father states that she began walking at 12 months. However, more recently, she has been losing her balance and has been falling. On physical exam, she has truncal ataxia and an ataxic gait. An ocular exam reveals dilation of the retinal capillaries. Labs are ordered that show an elevated alpha-fetoprotein. Which of the following is the most likely diagnosis?
Which of the following is most commonly associated with NMDA-receptor encephalitis?
Which of the following is the mechanism of action for selegiline?
A 57-year-old woman with a past medical history of hypothyroidism was referred to the EMG lab with a one-year history of right-hand numbness. Based on the EMG data shown below, what is the most appropriate next step in medical management?
A patient with relapsing-remitting multiple sclerosis presents to the emergency department with a new headache, confusion, and vision loss. A T2 sequence from their MRI is shown below. Lumbar puncture is positive for JCV in the CSF. What is the mechanism of action of the disease-modifying therapy most likely to be attributed to this clinical scenario?
Which of the following is not a risk factor for suicide?
Which is the following potentials on EMG has an irregular firing pattern?